Merge remote branch 'public/master'
[course.git] / latex / problems / Serway_and_Jewett_4 / problem14.05.tex
1 \begin{problem*}{14.5}
2 Two traveling sinusoidal waves are described by the wave functions
3 \begin{equation}
4   y_1 = (5.00\U{m})\cdot\sin[\pi(4.00x-1200t)]
5 \end{equation}
6 and
7 \begin{equation}
8   y_2 = (5.00\U{m})\cdot\sin[\pi(4.00x-1200t-0.250)]
9 \end{equation}
10 where $x$, $y_1$, and $y_2$ are in meters and $t$ is in seconds.
11 \Part{a} What is the amplitude of the resultant wave?
12 \Part{b} What is the frequency of the resultant wave?
13 \end{problem*} % problem 14.5
14
15 \begin{solution}
16 \Part{a}
17 \begin{equation}
18   y = y_1 + y_2
19     = 5.00\U{m}\cdot\p\{{\sin[\pi(4.00x-1200t)]+\sin[\pi(4.00x-1200t-0.250)}\}
20     = 5.00\U{m}\cdot\p[{\sin(\theta)+\sin(\theta-\pi/4)}] \;,
21 \end{equation}
22 where $\theta\equiv\pi(4.00x-1200t)$.  So $y_2$ trails $y_1$ by
23 $\pi/4=45\dg$.  In terms of the reference circle, that looks like
24 \begin{center}
25 \begin{asy}
26 import Mechanics;
27
28 real u = 1.2cm;
29 pair p = (u,0);
30 pair q = rotate(-45)*p;
31
32
33 draw(scale(u)*unitcircle, dotted);
34
35 Angle a = Angle(p, (0,0), q, radius=6mm, "$\pi/4$");
36 a.draw();
37 draw(p--(p+q), blue+dashed);
38 draw(q--(p+q), red+dashed);
39 draw((0,0)--p, red);
40 draw((0,0)--q, blue);
41 draw((0,0)--(p+q), green);
42 dot(p);
43 dot(q);
44 dot(p+q);
45 \end{asy}
46 \end{center}
47 The amplitude of $y$ is then given by vector addition
48 \begin{equation}
49   A = 2\cdot A\cos(\phi/2)
50     = 2\cdot 5.00\U{m}\cdot\cos(\pi/8) = \ans{9.24\U{m}} \;,
51 \end{equation}
52 where $\phi=\pi/4$ is the phase difference between $y_1$ and $y_2$.
53
54 \Part{b}
55 Both $y_1$ and $y_2$ rotate around the reference circle with a frequency of
56 \begin{equation}
57   f = \frac{\omega}{2\pi} = \frac{1200\pi\U{rad/s}}{2\pi\U{rad/cycle}}
58     = 600\U{Hz} \;,
59 \end{equation}
60 so $y$ must also rotate around the reference circle at $\ans{600\U{Hz}}$.
61 \end{solution}